Đến nội dung

Hình ảnh

[MSS2013] - Trận 20 - Bất đẳng thức


  • Chủ đề bị khóa Chủ đề bị khóa
Chủ đề này có 27 trả lời

#1
E. Galois

E. Galois

    Chú lùn thứ 8

  • Quản lý Toán Phổ thông
  • 3861 Bài viết

Chuyển nhanh đến:
1) Điều lệ
2) Đăng kí thi đấu
3) Lịch thi đấu và tổng hợp kết quả


Vào hồi 20h, Thứ Sáu, ngày 22/02/2013, Tổ trọng tài sẽ ra đề vào topic này, sau khi có đề, các toán thủ bắt đầu thi đấu.

Các toán thủ khi thi đấu, cứ yên tâm rằng, sau khi trả lời là bài làm đã được lưu, BTC đã nhận được bài làm của bạn, có điều bạn không nhìn thấy được mà thôi. Bạn nên mừng vì điều này, như thế các toán thủ khác không thể copy bài của bạn được.

Bạn cũng nên sử dụng chức năng xem trước của diễn đàn để sửa các lỗi Latex trước khi gửi bài, vì gửi rồi sẽ không xem và sửa lại được nữa.

BTC lưu ý:

1) Các toán thủ chớ quên rằng mỗi một mở rộng đúng sẽ được 10 điểm, các bạn nên mở rộng bài toán để thu được nhiều điểm hơn.

2) Sau khi trận đấu kết thúc, toán thủ nào tự ý sửa bài làm của mình sẽ được 0 điểm

3) Trận 20 có 31 toán thủ thi đấu nên sẽ có 11 toán thủ bị loại

1) Xem cách đăng bài tại đây
2) Học gõ công thức toán tại: http://diendantoanho...oạn-thảo-latex/
3) Xin đừng đặt tiêu đề gây nhiễu: "Một bài hay", "... đây", "giúp tớ với", "cần gấp", ...
4) Ghé thăm tôi tại 
http://Chúlùnthứ8.vn

5) Xin đừng hỏi bài hay nhờ tôi giải toán. Tôi cực gà.


#2
E. Galois

E. Galois

    Chú lùn thứ 8

  • Quản lý Toán Phổ thông
  • 3861 Bài viết
Cho a,b,c>0 thoả mãn: $2a+2b+c\geq 6$ & $a\leq b$. Tìm giá trị nhỏ nhất của biểu thức:
$$P=\sqrt{a^2+b^2}+\sqrt{c^2+4b^2}+\sqrt{4(a^2+b^2)+c^2}+a^4+b^3+c^2+b-2c$$
Đề của Nguyen Duc Thuan

1) Xem cách đăng bài tại đây
2) Học gõ công thức toán tại: http://diendantoanho...oạn-thảo-latex/
3) Xin đừng đặt tiêu đề gây nhiễu: "Một bài hay", "... đây", "giúp tớ với", "cần gấp", ...
4) Ghé thăm tôi tại 
http://Chúlùnthứ8.vn

5) Xin đừng hỏi bài hay nhờ tôi giải toán. Tôi cực gà.


#3
Tru09

Tru09

    Thiếu úy

  • Thành viên
  • 625 Bài viết

Cho a,b,c>0 thoả mãn: $2a+2b+c\geq 6$ & $a\leq b$. Tìm giá trị nhỏ nhất của biểu thức:
$$P=\sqrt{a^2+b^2}+\sqrt{c^2+4b^2}+\sqrt{4(a^2+b^2)+c^2}+a^4+b^3+c^2+b-2c$$
Đề của Nguyen Duc Thuan

Bài làm :
@) Xét : $a^4 +b^3 +c^2 +b -2c $
Ta có ( Sử dụng Cauchy )
$a^4 +1+1+1 \geq 4a$
$b^3 +1+1 \geq 3b$
$c^2 +4 \geq 4c$
Vậy
$\Rightarrow a^4 +b^3 +c^2 +9 \geq 4a+3b+4c$
$\Rightarrow a^4 +b^3 +c^2 +b -2c +9 \geq 4a+3b+4c +b-2c$
$\Rightarrow a^4 +b^3 +c^2 +b -2c +9\geq 4a+4b+2c \geq 12$
$\Rightarrow a^4 +b^3 +c^2 +b -2c \geq 3$
@) Xét : $\sqrt{a^2+b^2}+\sqrt{c^2+4b^2}+\sqrt{4(a^2+b^2)+c^2}$
Nhận xét :
---$2(x^2 +y^2) \geq (x+y)^2$
$\Leftrightarrow x^2 +y^2 \geq 2xy$
$\Leftrightarrow (x-y)^2 \geq 0 :\text{Luôn đúng}$
---$3(x^2+y^2+z^2) \geq (x+y+z)^2$
$\Leftrightarrow 2(x^2+y^2+z^2) \geq 2xy+2xz+2yz$
$\Leftrightarrow (x-y)^2 +(y-z)^2 +(z-x)^2 \geq 0 :\text{Luôn đúng}$
Ta có (áp dụng nhận xét )
$\sqrt{a^2+b^2} \geq \frac{a+b}{\sqrt{2}}$
$ \sqrt{c^2+4b^2} \geq \frac{c+2b}{\sqrt{2}}$
$\sqrt{4(a^2+b^2)+c^2} \geq \frac{2a+2b+c}{\sqrt{3}} \geq \frac{6}{\sqrt{3}}$
Vậy
$\sqrt{a^2+b^2}+\sqrt{c^2+4b^2}+\sqrt{4(a^2+b^2)+c^2} \geq \frac{a+3b+c}{\sqrt{2}} +\frac{6}{\sqrt{3}}$
$\Rightarrow \sqrt{a^2+b^2}+\sqrt{c^2+4b^2}+\sqrt{4(a^2+b^2)+c^2} \geq \frac{2a+2b+c}{\sqrt{2}} +\frac{6}{\sqrt{3}} $ :$\text{Do $b \geq a$}$
$\Rightarrow \sqrt{a^2+b^2}+\sqrt{c^2+4b^2}+\sqrt{4(a^2+b^2)+c^2} \geq \frac{6}{\sqrt{2}} +\frac{6}{\sqrt{3}}$
Cộng Lại ta có :
$P \geq \frac{6}{\sqrt{2}} +\frac{6}{\sqrt{3}} +3=3\sqrt{2} +2\sqrt{3} +3 $
Vậy $P_{min} = 3\sqrt{2} +2\sqrt{3} +3$
Dấu $=$ xảy ra $\Leftrightarrow a=b=1 , c=2$

__________________________________
@Joker: Kết quả chính xác, lời giải không có sai sót nào. Tuy nhiên lần sau nên viết nhận xét thành bổ đề để bài viết được sáng sủa hơn.
Chấm điểm: d=10

S = 26 + 3*10 +10 + 10= 76

Bài viết đã được chỉnh sửa nội dung bởi E. Galois: 01-03-2013 - 17:56
Ghi điểm


#4
daovuquang

daovuquang

    Trung sĩ

  • Thành viên
  • 194 Bài viết
Bài làm của daovuquang:
Bổ đề 1: BĐT Cauchy. Có thể xem c/m ở đây:http://vi.wikipedia....ẳng_thức_Cauchy
Bổ đề 2: Với $a,b,c$ dương, ta có: $a^2+b^2+c^2\geq \frac{(a+b+c)^2}{3}\; (*)$
Chứng minh: $(*) \Leftrightarrow 3(a^2+b^2+c^2)\geq (a^2+b^2+c^2+2ab+2bc+2ca)$
$\Leftrightarrow (a-b)^2+(b-c)^2+(c-a)^2\geq 0$.
Bđt cuối luôn đúng.
Dấu đẳng thức xảy ra $\Leftrightarrow a=b=c$.

Quay trở lại bài toán:
Nhận thấy: $(a-b)^2 \geq 0 $
$\Leftrightarrow a^2+b^2\geq 2ab$
$\Leftrightarrow a^2+b^2 \geq \frac{(a+b)^2}{2}$
$\Leftrightarrow \sqrt{a^2+b^2} \geq \frac{a+b}{\sqrt{2}}$.
Tương tự, $\sqrt{4b^2+c^2}\geq \frac{2b+c}{\sqrt{2}}$
$\Leftrightarrow \sqrt{a^2+b^2}+\sqrt{4b^2+c^2}\geq \frac{a+3b+c}{\sqrt{2}}\geq \frac{2a+2b+c}{\sqrt{2}}\geq 3\sqrt{2}$.($1$) ( Chỗ này cần nên ghi $b\geq a$ )
Áp dụng bổ đề 2, ta có: $\sqrt{4(a^2+b^2)+c^2}\geq \sqrt{\frac{(2a+2b+c)^2}{3}}=\frac{2a+2b+c}{\sqrt{3}}\geq 2\sqrt{3}\; (2)$.
Áp dụng bổ đề 1, ta có:

$\left\{\begin{matrix}
a^4+1+1+1\geq 4a\\
b^3+1+1\geq 3b\\
c^2+4\geq 4c
\end{matrix}\right.$
$\Rightarrow a^4+b^3+c^2+b-2c\geq 2(2a+2b+c)-9\geq 3\; (3)$.
Từ $(1),(2),(3) \Rightarrow P \geq 2\sqrt{3}+3\sqrt{2}+3$.
Dấu đẳng thức xảy ra $\Leftrightarrow a=b=1$ và $c=2$.
Vậy GTNN của $P=2\sqrt{3}+3\sqrt{2}+3$. Dấu đẳng thức xảy ra $\Leftrightarrow a=b=1$ và $c=2$.
_____________________________________________
@Joker: Lời giải về cơ bản là chính xác. Đáp số đúng. Tuy nhiên về CM BDT AM-GM thì có lẽ không cần ghi ra, và nếu muốn chứng minh 1 bổ đề nào đó thì bạn nên chứng minh cụ thể chứ không nên ghi xem ở đây, xem tại link sau ....
Chấm điểm:d=10 -( Tuy chưa được hoàn hảo )

S = 25 + 10*3 + 10 + 10= 75

Bài viết đã được chỉnh sửa nội dung bởi E. Galois: 01-03-2013 - 17:34
Chấm bài


#5
BlackSelena

BlackSelena

    $\mathbb{Sayonara}$

  • Hiệp sỹ
  • 1549 Bài viết

Cho a,b,c>0 thoả mãn: $2a+2b+c\geq 6$ & $a\leq b$. Tìm giá trị nhỏ nhất của biểu thức:
$$P=\sqrt{a^2+b^2}+\sqrt{c^2+4b^2}+\sqrt{4(a^2+b^2)+c^2}+a^4+b^3+c^2+b-2c$$
Đề của Nguyen Duc Thuan

Bài làm của MSS01 - BlackSelena
Trước hết ta có bất đẳng thức phụ sau:
$a^2+b^2+c^2 \geq \frac{(a+b+c)^2}{3}$
$\Leftrightarrow (a-b)^2 + (b-c)^2 + (c-a)^2 \geq 0: \text{ luôn đúng}$, đẳng thức xảy ra khi $a=b=c$
Lần lượt áp dụng bdt $AM-GM$, ta có:
$\sqrt{a^2+b^2}\geq \frac{a+b}{\sqrt{2}}
\\
\sqrt{c^2+4b^2}\geq \frac{c+2b}{\sqrt{2}}
\\
\sqrt{4(a^2+b^2)+c^2} = \sqrt{(2a)^2 + (2b)^2 + c^2} \geq \frac{2a+2b+c}{\sqrt{3}}
\\
a^4 + 1 + 1 + 1 \geq 4a
\\
b^3 + 1 + 1 \geq 3b
\\
c^2 + 4 \geq 2c$
Vậy ta có
$ P + 9 \geq \dfrac{a+b+c+2b}{\sqrt{2}} + \dfrac{2a+2b+c}{\sqrt{3}} + 4a + 3b + 2c +b - 2c$
$\geq \dfrac{2a +2b + c}{\sqrt{2}} + \dfrac{2a+2b+c}{\sqrt{3}} + 2(2a+2b+c) $ (do $a \leq b$)
$\geq \dfrac{6}{\sqrt{2}} + \dfrac{6}{\sqrt{3}} + 12 = 3\sqrt{2} + 2\sqrt{3} + 12$ (do $2a+2b+c \geq 6$)
$\Rightarrow P \geq 3\sqrt{2} + 2\sqrt{3} + 3$
Vậy $\min P = 3\sqrt{2} + 2\sqrt{3} + 3$; đẳng thức xảy ra khi $a=b=1; c=2$
________________________________
@Joker: Lời giải chính xác, đáp số đúng, trình bày gọn gàng
Chấm điểm: d=10

S = 25 + 10*3 + 10 + 10 = 75

Bài viết đã được chỉnh sửa nội dung bởi E. Galois: 01-03-2013 - 21:57
Chấm bài


#6
The gunners

The gunners

    Binh nhì

  • Thành viên
  • 17 Bài viết
Áp dụng BDT Bunhiacopxki:
$\sqrt{a^2+b^2}+\sqrt{4b^2+c^2}+\sqrt{4a^2+4b^2+c^2}\geq \frac{a+b}{\sqrt{2}}+\frac{b+2c}{\sqrt{2}}+\frac{2a+2b+2c}{\sqrt{3}}=\frac{a+3b+c}{\sqrt{2}}+\frac{6}{\sqrt{3}}$ ( Nhầm ở đây)
Mà:
$\frac{a+3b+c}{\sqrt{2}}+\frac{6}{\sqrt{3}}\geq \frac{2a+2b+c}{\sqrt{2}}+\frac{6}{\sqrt{3}}$ ( Do $a\geq b$)
Nên
$\sqrt{a^2+b^2}+\sqrt{4b^2+c^2}+\sqrt{4a^2+4b^2+c^2}\geq \frac{6}{\sqrt{2}}+\frac{6}{\sqrt{3}}$ (1)
Mặt khác theo BDT Cauchy ta có:
$a^4\geq 4a-3$ (2) ( Chỗ này nên ghi rõ $a^4+1+1+1\geq 4a$)
$b^3\geq 3b-2$ (3)
$c^2\geq 4c-4$ (4)
Từ (1)(2)(3) và (4) suy ra:
P$\geq \frac{6}{\sqrt{2}}+\frac{6}{\sqrt{3}}+(4a-3)+(3b-2)+(4c-4)+b-2c=3+3\sqrt{2}+2\sqrt{3}$ ( do 2a+2a+c=6 ) phải là $2a+2b+c\geq 6$
Đẳng thức xảy ra khi và chỉ khi: a=b=1 ; c=2
Vậy minP=$3+3\sqrt{2}+2\sqrt{3}$$\Leftrightarrow$ a=b=1;c=2
____________________________
@Joker: Đáp số đúng.
Chấm điểm: d=8,5

@hxthanh: Điểm: d=8 (Điểm nguyên nhé em!)

S = 25 + 3*8 = 49

Bài viết đã được chỉnh sửa nội dung bởi E. Galois: 01-03-2013 - 22:09
Chấm bài


#7
nguyenqn1998

nguyenqn1998

    Trung sĩ

  • Thành viên
  • 173 Bài viết
Ta có
P=$\sqrt{a^2+b^2}$ + $\sqrt{c^2+4b^2}$ + $\sqrt{4(a^2+b^2)+c^2}$ + a^4 + b^3 +c^2 + b -2c \geq \frac{a+b}{\sqrt{2}} + \frac{c+2b}{\sqrt{2}} + \frac{2a+2b+c}{\sqrt{3}} + a^4+1+1+1 + b^3 +1+1 +2c +b +c^2 -4c+4 -9 \geq \frac{2a+2b+c}{\sqrt{2}} + 2\sqrt{3} + 4\sqrt[4]{a^4}+\sqrt[3]{b^3} +2c +b + (c-2)^2 -9 = 3\sqrt{2}+2\sqrt{3} + 2(2a+2b+c) + (c-2)^2-9\geq 3\sqrt{2}+2\sqrt{3} + 3$
Vậy Min P= 3\sqrt{2}+2\sqrt{3} + 3 dấu ''='' xảy ra khi a=b=1; c=2
_________
@Joker: Lỗi Latex
d=1

Bài viết đã được chỉnh sửa nội dung bởi E. Galois: 01-03-2013 - 17:29
Chấm bài


#8
nguyenqn1998

nguyenqn1998

    Trung sĩ

  • Thành viên
  • 173 Bài viết
Ta có:
P=$$\sqrt{a^2+b^2}$ + $\sqrt{c^2+4b^2}$ + $\sqrt{4(a^2+b^2)+c^2}$ + a^4 + b^3 +c^2 + b -2c \geq \frac{a+b}{\sqrt{2}} + \frac{c+2b}{\sqrt{2}} + \frac{2a+2b+c}{\sqrt{3}} + a^4+1+1+1 + b^3 +1+1 +2c +b +c^2 -4c+4 -9 \geq \frac{2a+2b+c}{\sqrt{2}} + 2\sqrt{3} + 4\sqrt[4]{a^4}+3\sqrt[3]{b^3} +2c +b + (c-2)^2 -9 = 3\sqrt{2}+2\sqrt{3} + 2(2a+2b+c) + (c-2)^2-9\geq 3\sqrt{2}+2\sqrt{3} + 3$
Vậy MinP= 3\sqrt{2}+2\sqrt{3} + 3 dấu ''='' xảy ra khi a=b=1;c=2

#9
nhatquangsin

nhatquangsin

    Thượng sĩ

  • Thành viên
  • 238 Bài viết
Snap2.png $Ta có: $\sqrt{x^{2}+y^{2}}\geq \frac{x+y}{\sqrt{2}}$. Nên: P$\geq \frac{a+b}{\sqrt{2}}+\frac{c+2b}{\sqrt{2}}+\frac{2a+2b+2c}{\sqrt{3}}+a^{4}+b^{3}+c^{2}+b-2c\geq \frac{2a}{\sqrt{2}}+\frac{c+2b}{\sqrt{2}}+\frac{6}{\sqrt{3}}+a^{4}+b^{3}+c^{2}+b-2c\geq \frac{6}{\sqrt{2}}+\frac{6}{\sqrt{3}}+a^{4}+b^{3}+c^{2}+b-2c.$. Lại có: $a^{4}+1+1+1\geq 4a/ b^{3}+b+1+1\geq 4b/ c^{2} -2c +4\geq 2c.$Nên: $a^{4}+b^{3}+c^{2}+b-2c+9\geq 4a+4b+2c\geq 12\Leftrightarrow a^{4}+b^{3}+c^{2}+b-2c\geq 3$$\Rightarrow P\geq \frac{6}{\sqrt{2}}+\frac{6}{\sqrt{3}}+3$. Dấu bằng khi 2a=2b=c và 2a+2b+c=6$\Leftrightarrow$ a=b=1, c=2
__________
@Joker: Lỗi latex. Nên sử dụng chức năng xem trước.
d=1

Bài viết đã được chỉnh sửa nội dung bởi E. Galois: 01-03-2013 - 17:29
Chấm bài


#10
LNH

LNH

    Bất Thế Tà Vương

  • Hiệp sỹ
  • 581 Bài viết
Bài làm của MSS50-lenhathoang1998
Đầu tiên, ta chứng minh bất đẳng thức AM-GM n biến:
Xem trên http://vi.wikipedia....ẳng_thức_Cauchy
Trở lại bài toán:
Ta có:
$\sqrt{{{a}^{2}}+{{b}^{2}}}\ge \frac{a+b}{\sqrt{2}}\ge \frac{2a}{\sqrt{2}}$ (Áp dụng BDT Cauchy-Schwatrz 2 biến và dữ kiện $a\le b$)
$\sqrt{{{c}^{2}}+4{{b}^{2}}}\ge \frac{c+2b}{\sqrt{2}}$
$\sqrt{4{{a}^{2}}+4{{b}^{2}}+{{c}^{2}}}\ge \frac{2a+2b+c}{\sqrt{3}}$ (Áp dụng BDT Cauchy-Schwatrz 3 biến)
${{a}^{4}}+3\ge 4a$ (AM-GM 4 biến)
${{b}^{3}}+b+2\ge 4b$ (AM-GM 4 biến)
${{c}^{2}}-2c+4\ge 2c$ (BDT tương đương ${{(c-2)}^{2}}\ge 0$ )
$\Rightarrow P\ge \frac{2a}{\sqrt{2}}+\frac{2b+c}{\sqrt{2}}+\frac{2a+2b+c}{\sqrt{3}}+4a+4b+c-3-2-4\ge \frac{6}{\sqrt{2}}+\frac{6}{\sqrt{3}}+12-3-2-4=3\sqrt{2}+2\sqrt{3}+3$ ( Phải là $4a+4b+2c$)
$\Rightarrow \min P=3\sqrt{2}+2\sqrt{3}+3$
Khi đó:
$a=b=1$
$c=2$
________________________________________________--
@Joker: Lời giải có đáp số đúng, tương đối chính xác. Có $1$ chỗ viết nhầm. BDT AM-GM có lẽ không cần chứng minh lại
Chấm điểm : d=9.

S = 13 + 3*9 = 40

Bài viết đã được chỉnh sửa nội dung bởi E. Galois: 01-03-2013 - 17:31
Ghi điểm


#11
Tru09

Tru09

    Thiếu úy

  • Thành viên
  • 625 Bài viết
Mở rộng 1 :Cho $a,b,c >0$ thỏa mãn $2a +2b+c \geq 10 $ & $a \leq b$ Tìm giá trị nhỏ nhất của biểu thức :
P = $\sqrt{a^2+b^2}+\sqrt{c^2+4b^2}+\sqrt{4(a^2+b^2)+c^2} +a^4 +b^3 +c^2 +b -2c -\frac{248a}{27}$
Bài làm :

@) Xét : $a^4 +b^3 +c^2 +b -2c $
Ta có ( Sử dụng Cauchy )
$a^4 +(\frac{10}{6})^4+(\frac{10}{6})^4+(\frac{10}{6})^4 \geq 4a.(\frac{10}{6})^3$
$b^3 +(\frac{10}{6})^3+(\frac{10}{6})^3 \geq 3b(\frac{10}{6})^2$
$c^2 +(\frac{10}{3})^2 \geq 2c.\frac{10}{3}$
Vậy
$\Rightarrow a^4 +b^3 +c^2 +3(\frac{10}{6})^4 +2(\frac{10}{6})^3 +(\frac{10}{3})^2 \geq 4a.(\frac{10}{6})^3+3b(\frac{10}{6})^2+2c.\frac{10}{3}$
$\Rightarrow a^4 +b^3 +c^2 +b -2c +3(\frac{10}{6})^4 +2(\frac{10}{6})^3 +(\frac{10}{3})^2 -\frac{248a}{27} \geq 4a.(\frac{10}{6})^3+3b(\frac{10}{6})^2+2c.\frac{10}{3}+b-2c -\frac{248a}{27}$
$\Rightarrow a^4 +b^3 +c^2 +b -2c +3(\frac{10}{6})^4 +2(\frac{10}{6})^3 +(\frac{10}{3})^2 -\frac{248a}{27} \geq \frac{28}{3}(a+b+\frac{c}{2}) \geq \frac{28}{3} .5 =\frac{140}{3}$
$\Rightarrow a^4 +b^3 +c^2 +b -2c-\frac{248a}{27} \geq \frac{85}{27}$
@) Xét : $\sqrt{a^2+b^2}+\sqrt{c^2+4b^2}+\sqrt{4(a^2+b^2)+c^2}$
Nhận xét :
---$2(x^2 +y^2) \geq (x+y)^2$
$\Leftrightarrow x^2 +y^2 \geq 2xy$
$\Leftrightarrow (x-y)^2 \geq 0 :\text{Luôn đúng}$
---$3(x^2+y^2+z^2) \geq (x+y+z)^2$
$\Leftrightarrow 2(x^2+y^2+z^2) \geq 2xy+2xz+2yz$
$\Leftrightarrow (x-y)^2 +(y-z)^2 +(z-x)^2 \geq 0 :\text{Luôn đúng}$
Ta có (áp dụng nhận xét )
$\sqrt{a^2+b^2} \geq \frac{a+b}{\sqrt{2}}$
$ \sqrt{c^2+4b^2} \geq \frac{c+2b}{\sqrt{2}}$
$\sqrt{4(a^2+b^2)+c^2} \geq \frac{2a+2b+c}{\sqrt{3}} \geq \frac{10}{\sqrt{3}}$
Vậy
$\sqrt{a^2+b^2}+\sqrt{c^2+4b^2}+\sqrt{4(a^2+b^2)+c^2} \geq \frac{a+3b+c}{\sqrt{2}} +\frac{10}{\sqrt{3}}$
$\Rightarrow \sqrt{a^2+b^2}+\sqrt{c^2+4b^2}+\sqrt{4(a^2+b^2)+c^2} \geq \frac{2a+2b+c}{\sqrt{2}} +\frac{10}{\sqrt{3}} $ :$\text{Do $b \geq a$}$
$\Rightarrow \sqrt{a^2+b^2}+\sqrt{c^2+4b^2}+\sqrt{4(a^2+b^2)+c^2} \geq \frac{10}{\sqrt{2}} +\frac{10}{\sqrt{3}}$
Cộng Lại ta có :
$P \geq \frac{10}{\sqrt{2}} +\frac{10}{\sqrt{3}} +\frac{85}{27}$
Vậy $P_{min} = \frac{10}{\sqrt{2}} +\frac{10}{\sqrt{3}} +\frac{85}{27}$
Dấu $=$ xảy ra $\Leftrightarrow a=b=\frac{5}{3} , c=\frac{10}{3}$

Điểm mở rộng 10

Bài viết đã được chỉnh sửa nội dung bởi E. Galois: 01-03-2013 - 17:36
Ghi điểm


#12
NguyenThinhTin

NguyenThinhTin

    Lính mới

  • Thành viên
  • 2 Bài viết
Ta chứng minh các bất đẳng thức phụ:
$2(a^{2}+b^{2})\geq (a+b)^{2}$ (1)
Từ (1) $\Rightarrow$ $2a^{2}+2b^{2} \geq a^{2} +2ab +b^{2} \Leftrightarrow a^{2}+b^{2}\geq 2ab (*)$ ( Không phải là suy ra mà là tương đương )
vì a,b$>$0 nên (*) luôn đúng (BĐT Cô-si) do đó (1) luôn đúng Dấu "=" xảy ra khi và chỉ khi a=b
$3(a^{2}+b^{2}+c^{2})\geq (a+b+c)^{2}$ (2)
Từ (2) suy ra: $3a^{2}+3b^{2}+3c^{2}\geq a^{2}+b^{2}+c^{2}+2ab+2bc+2ca$
Hay $2a^{2}+2b^{2}+2c^{2}-2ab-2bc-2ca\geq 0 \Leftrightarrow (a-b)^{2}+(b-c)^{2}+(c-a)^{2}\geq 0$ (**)
Vì (**) luôn đúng nên (2) luôn đúng Dấu "=" xảy ra khi và chỉ khi a=b=c
$a+b+c+d\geq 4\sqrt[4]{abcd}$ (BĐT Cô-si mở rộng cho 4 hạng tử với a,b,c,d $>$ 0) (3)
Ta có: Vì a,b,c $>$ 0 nên:
Từ (1) cho ta: $\sqrt{a^{2}+b^{2}}\geq \frac{a+b}{^{\sqrt{2}}}$
Từ (1) cho ta: $\sqrt{c^{2}+4b^{2}}\geq \frac{c+2b}{^{\sqrt{2}}}\geq \frac{c+a+b}{\sqrt{2}}$ (vì a$\leq$b
Từ (2) cho ta: $\sqrt{4(a^{2}+b^{2})+c^{2}}\geq \frac{2a+2b+c}{\sqrt{3}}$=$\frac{6}{\sqrt{3}}$ ( Dấu lớn hơn hoặc bằng nhé)
Từ (3) cho ta : $a^{4}+3=a^{4}+1+1+1\geq 4\sqrt[4]{a}=4a$
Từ (3) cho ta: $b^{3}+b+2=b^{3}+b+1+1\geq 4\sqrt[4]{b^{3}b}=4b$
$c^{2}-2c+4\geq 2c$
Do đó: $a^{4}+b^{3}+c^{2}+b-2c=(a^{4}+3)+(b^{3}+b+2)+(c^{2}-2c+4)-9\geq 4a+4b+2c-9=2(2a+2b+c)-9=3$
Từ đó ta có P$\geq \frac{a+b}{\sqrt{2}}+\frac{c+a+b}{\sqrt{2}}+ \frac{6}{\sqrt{3}}+3=\frac{2a+2b+c}{\sqrt{2}}+\frac{6}{\sqrt{3}}+3=\frac{6}{\sqrt{2}}+\frac{6}{\sqrt{3}}+3\approx 10.7067423$ ( Nên để dưới dạng căn chứ không cần tính ra như vậy)
Vậy giá trị nhở nhất của P la 10.7067423 khi và chỉ khi
$\left\{\begin{matrix} a=b=1 & \\c=2 \end{matrix}\right.$
_______________________________
@Joker: Đáp số đúng. Viết nhầm vài chỗ
Chấm điểm d=9.

S = 12 + 3*9 = 39

Bài viết đã được chỉnh sửa nội dung bởi E. Galois: 01-03-2013 - 17:38
Ghi điểm


#13
daovuquang

daovuquang

    Trung sĩ

  • Thành viên
  • 194 Bài viết
Mở rộng 1: Cho a,b,c dương thỏa mãn $2a+2b+c\geq 6m$ với $m>0$ và $a \leq b$. Tìm GTNN của biểu thức:
$$P=\sqrt{a^2+b^2}+\sqrt{c^2+4b^2}+\sqrt{4(a^2+b^2)+c^2}+a^4+b^3+c^2+(4m^3-3m^2)b+(2m^3-4m)c$$

Tương tự bài trước, ta có:

$\left\{\begin{matrix}
\sqrt{a^2+b^2}\geq \frac{a+b}{\sqrt{2}}\\
\sqrt{c^2+4b^2}\geq \frac{c+2b}{\sqrt{2}}
\end{matrix}\right.$
$\Rightarrow \sqrt{a^2+b^2}+\sqrt{c^2+4b^2}\geq \frac{a+3b+c}{\sqrt{2}}\geq \frac{2a+2b+c}{\sqrt{2}}\geq 3\sqrt{2}m\; (1)$.
Mặt khác, $\sqrt{4(a^2+b^2)+c^2}\geq \frac{2a+2b+c}{\sqrt{3}}\geq 2\sqrt{3}m\; (2)$.
Áp dụng BĐT Cauchy, ta có:

$\left\{\begin{matrix}
a^4+m^4+m^4+m^4 \geq 4m^3a\\
b^3+m^3+m^3\geq 3m^2b\\
c^2+4m^2\geq 4mc
\end{matrix}\right.$
$\Rightarrow a^4+b^3+c^2+(4m^3-3m^2)b+(2m^3-4m)c\geq 4m^3a+(4m^3-3m^2+3m^2)b+(2m^3-4m+4m)c-3m^4-2m^3-4m^2$
$\Rightarrow a^4+b^3+c^2+(4m^3-3m^2)b+(2m^3-4m)c\geq 2m^3(2a+2b+c)-3m^4-2m^3-4m^2\geq 9m^4-2m^3-4m^2\; (3)$.
Từ $(1),(2),(3) \Rightarrow P \geq 2\sqrt{3}m+3\sqrt{2}m+9m^4-2m^3-4m^2$.
Dấu đẳng thức xảy ra $\Leftrightarrow a=b=m$ và $c=2m$.
Vậy GTNN của $P=2\sqrt{3}m+3\sqrt{2}m+9m^4-2m^3-4m^2 \Leftrightarrow a=b=m$ và $c=2m$.


Điểm mở rộng 10

Bài viết đã được chỉnh sửa nội dung bởi E. Galois: 01-03-2013 - 17:36
Ghi điểm


#14
daovuquang

daovuquang

    Trung sĩ

  • Thành viên
  • 194 Bài viết
Mở rộng 2: Cho $a,b,c$ dương thỏa mãn $2a+2b+c\geq 6m$ với $m>0$ và $a \leq b$. Với $x,y,z \in \mathbb{N}^*$ và $x,y,z\geq 2$, tìm GTNN của biểu thức:
$$P=\sqrt{a^2+b^2}+\sqrt{c^2+4b^2}+\sqrt{4(a^2+b^2)+c^2}+a^x+b^y+c^z+(xm^{x-1}-ym^{y-1})b+(\frac{x}{2}m^{x-1}-2^{z-1}zm^{z-1})c$$

Tương tự bài trước, ta có:

$\left\{\begin{matrix}
\sqrt{a^2+b^2}\geq \frac{a+b}{\sqrt{2}}\\
\sqrt{c^2+4b^2}\geq \frac{c+2b}{\sqrt{2}}
\end{matrix}\right.$
$\Rightarrow \sqrt{a^2+b^2}+\sqrt{c^2+4b^2}\geq \frac{a+3b+c}{\sqrt{2}}\geq \frac{2a+2b+c}{\sqrt{2}}\geq 3\sqrt{2}m\; (1)$.
Mặt khác, $\sqrt{4(a^2+b^2)+c^2}\geq \frac{2a+2b+c}{\sqrt{3}}\geq 2\sqrt{3}m\; (2)$.
Áp dụng BĐT Cauchy, ta có:

$\left\{\begin{matrix}
a^x+m^x+m^x+...+m^x \geq xm^{x-1}a\\
b^y+m^y+m^y+...+m^y\geq ym^{y-1}b\\
c^z+(2m)^z+(2m)^z+...+(2m)^z\geq 2^{z-1}zm^{z-1}c
\end{matrix}\right.$
$\Rightarrow a^x+b^y+c^z+(xm^{x-1}-ym^{y-1})b+(\frac{x}{2}m^{x-1}-2^{z-1}zm^{z-1})c\geq xm^{x-1}a+(xm^{x-1}-ym^{y-1}+ym^{y-1})b+(\frac{x}{2}m^{x-1}-2^{z-1}zm^{z-1}+2^{z-1}zm^{z-1})c-(x-1)m^x-(y-1)m^y-2^z(z-1)m^z$
$\Rightarrow a^x+b^y+c^z+(xm^{x-1}-ym^{y-1})b+(\frac{x}{2}m^{x-1}-2^{z-1}zm^{z-1})c\geq 3xm^x-(x-1)m^x-(y-1)m^y-2^z(z-1)m^z=(2x+1)m^x-(y-1)m^y-2^z(z-1)m^z\; (3)$.
Từ $(1),(2),(3) \Rightarrow P \geq 2\sqrt{3}m+3\sqrt{2}m+(2x+1)m^x-(y-1)m^y-2^z(z-1)m^z$.
Dấu đẳng thức xảy ra $\Leftrightarrow a=b=m$ và $c=2m$.
Vậy GTNN của $P=2\sqrt{3}m+3\sqrt{2}m+(2x+1)m^x-(y-1)m^y-2^z(z-1)m^z \Leftrightarrow a=b=m$ và $c=2m$.

Điểm mở rộng 10

Bài viết đã được chỉnh sửa nội dung bởi E. Galois: 01-03-2013 - 17:39
Ghi điểm


#15
field9298

field9298

    Hạ sĩ

  • Thành viên
  • 50 Bài viết
Ta có $b\geq a\Leftrightarrow \sqrt{a^{2}+b^{2}}\geq \sqrt{2a^{2}}= a\sqrt{2}$
Áp dụng bất đẳng thức Bunhiacopxki ta được:
$\sqrt{(c^{2}+4b^{2})(1^{2}+1^{2})}\geq c+2b\Leftrightarrow \sqrt{c^{2}+4b^{2}}\geq\frac{c+2b}{\sqrt{2}}$
$2(a^{2}+b^{2})\geq (a+b)^{2}$
$\sqrt{\left [2(a+b)^{2}+c^{2} \right ](1^{2}+1^{2}+1^{2})}\geq a+b+a+b+c\geq 6\Leftrightarrow \sqrt{2(a+b)^2+c^2}\geq 2\sqrt{3}$
Áp dụng bất đẳng thức Cô-si
Ta được: $a^{4}+1+1+1\geq 4\sqrt[4]{a^{4}.1.1.1}\Leftrightarrow 4a$
$b^{3}+b+1+1\geq 4\sqrt[4]{b^{3}.b.1.1}\Leftrightarrow b^{3}+b+2\geq 4b$
$c^2+4\geq 2\sqrt{4c^{2}}=4c$
Nên $P=\sqrt{a^{2}+b^{2}}+\sqrt{c^{2}+4b^{2}}+\sqrt{4(a^{2}+b^{2})+c^{2}}+a^{4}+b^{3}+c^{2}-2c$
$\geq a\sqrt{2}+\frac{c+2b}{\sqrt{2}}+\sqrt{2(a+b)^{2}+c^{2}}+(a^{4}+3)+(b^3+b+2)+(c^2+4)-2c-9$
$\geq \frac{2a+2b+c}{\sqrt{2}}+2\sqrt{3}+4a+4b+4c-2c-6$
$\geq \frac{6}{\sqrt{2}}+2\sqrt{3}+2(2a+2b+c)-9$
$\geq 3\sqrt{2}+2\sqrt{3}+12-9=3\sqrt{2}+2\sqrt{3}+3$
Vậy $PMin=3\sqrt{2}+2\sqrt{3}+3$ khi a=b=1;c=2
________________
@Joker: Về cơ bản là chính xác. Đoạn cuối có Latex nhỏ
Chấm điểm: d=10
S = 4 + 10*3 + 10= 44

Bài viết đã được chỉnh sửa nội dung bởi E. Galois: 01-03-2013 - 21:54
Ghi điểm


#16
field9298

field9298

    Hạ sĩ

  • Thành viên
  • 50 Bài viết
Mở rộng 1:Cho a,b,c>0 thoả mãn: 2a+2b+c6 & ab. Tìm giá trị nhỏ nhất của biểu thức:$P=\sqrt{a^{2}+b^{2}}+\sqrt{c^2+4b^2}+\sqrt{4(a^2+b^2)+c^2}+a^{4k}+b^{3k}+c^{2}.k+b^k-2kc$
Tương tự phần bài làm,ta chứng minh được $\sqrt{a^{2}+b^{2}}+\sqrt{c^2+4b^2}+\sqrt{4(a^2+b^2)+c^2}\geq 2\sqrt{3}+3\sqrt{2}$ và
Ta chỉ cần tìm GTNN của $B=a^{4k}+b^{3k}+c^{2}.k+b^k-2kc$
Áp dụng bất đẳng thức Cô-si ta được:$a^{4k}+1.(4k-1)\geq 4k\sqrt[4k]{a^{4k}}=4ka$
$b^{3k}+b^{k}+(4k-2).1\geq 4k\sqrt[4k]{b^{3k}.b^{k}}=4kb$
Chứng minh tương tự như bài làm ta được: $c^{2}+4\geq 4c\Leftrightarrow c^{2}-2c\geq 2c-4\Leftrightarrow k.c^{2}-2kc\geq 2kc-4k$
Suy ra $B=a^{4k}+b^{3k}+c^{2}.k+b^k-2kc=(a^{4k}+4k-1)+(b^{3k}+b^k+4k-2)+(c^{2}.k-2kc)+3-8k\geq 4ka+4kb+2kc+3-4k-8k=2k(2a+2b+c)+3-12k\geq 12k+3-12k=3$
Nên $P\geq 2\sqrt{3}+3\sqrt{2}+3$
$BMin=2\sqrt{3}+3\sqrt{2}+3$ khi a=b=1;c=2

Điểm mở rộng: 10

Bài viết đã được chỉnh sửa nội dung bởi E. Galois: 01-03-2013 - 21:51
Chấm điểm


#17
BlackSelena

BlackSelena

    $\mathbb{Sayonara}$

  • Hiệp sỹ
  • 1549 Bài viết
Mở rộng 1:
Giữ nguyên phần căn thức, đổi điều kiện thành $2a + 2b + c \geq 5$
Ta tổng quát số mũ của $a,b,c$ ở phần sau. Tức là tìm min:
$P = a^{2n} + b^{2n-1} + c^{2n-2} + b + (2-n)c + \sqrt{a^2+b^2} + \sqrt{c^2+4b^2} + \sqrt{4(a^2+b^2) + c^2}$
Phần: $\sqrt{a^2+b^2} + \sqrt{c^2+4b^2} + \sqrt{4(a^2+b^2) + c^2}$ thì làm như đề bài
Áp dụng bdt $AM-GM$, ta có:
$a^{2n} + (2n-1) \geq 2na$
$b^{2n-1} + (2n-2) \geq (2n-1)b$
$c^{2n-2} + (2n-3) \geq (2n-2)c$
Vậy $P + 2n -1 + 2n - 2 + 2n - 3 \geq \frac{5}{\sqrt{2}} + \dfrac{5}{\sqrt{3}} + 2na + 2nb -b + 2nc - 2c + b + 2c - nc = 2\sqrt{3} + 3\sqrt{2} + n(2a+2b+c) \geq 2\sqrt{3} + 3\sqrt{2} + 5n$
$\Rightarrow P \geq \dfrac{5}{\sqrt{2}} + \dfrac{5}{\sqrt{3}} + 6 - n$, đẳng thức xảy ra khi $a=b = 1; c = 1$

Điểm mở rộng 10

Bài viết đã được chỉnh sửa nội dung bởi E. Galois: 01-03-2013 - 17:42
Ghi điểm


#18
Tru09

Tru09

    Thiếu úy

  • Thành viên
  • 625 Bài viết
Mở rộng 2 :
Cho a,b,c>0 thoả mãn: $2a+2b+c\geq 6$ & $a\leq b$. Tìm giá trị nhỏ nhất của biểu thức:
$$P=\sqrt[3]{a^{3}+b^{3}}+\sqrt[3]{c^{3}+8b^{3}}+\sqrt[3]{8(a^{3}+b^{3})+c^{3}}+a^4+b^3+c^2+b-2c$$
@) Xét : $a^4 +b^3 +c^2 +b -2c $
Ta có ( Sử dụng Cauchy )
$a^4 +1+1+1 \geq 4a$
$b^3 +1+1 \geq 3b$
$c^2 +4 \geq 4c$
Vậy
$\Rightarrow a^4 +b^3 +c^2 +9 \geq 4a+3b+4c$
$\Rightarrow a^4 +b^3 +c^2 +b -2c +9 \geq 4a+3b+4c +b-2c$
$\Rightarrow a^4 +b^3 +c^2 +b -2c +9\geq 4a+4b+2c \geq 12$
$\Rightarrow a^4 +b^3 +c^2 +b -2c \geq 3$
@) Xét : $\sqrt[3]{a^{3}+b^{3}}+\sqrt[3]{c^{3}+8b^{3}}+\sqrt[3]{8(a^{3}+b^{3})+c^{3}}$
Nhận xét :
---$4(a^3+b^3) \geq (a+b)^3$
$\Leftrightarrow a^3+b^3 \geq ab(a+b)$
$\Leftrightarrow (a+b)(a-b)^2 \geq 0 :\text{Luôn đúng với $a ,b \geq 0$}$
Dấu $=$ khi $a =b$
---$9(a^3+b^3+c^3) \geq (a+b+c)^3$
$\Leftrightarrow 8(a^3+b^3+c^3) \geq 3(a+b)(b+c)(c+a) $
$\Leftrightarrow 6(a^3+b^3+c^3) +2(a^3+b^3+c^3)\geq 3ab(a+b) +3bc(b+c) +3ca(c+a) +6abc$
$\Leftrightarrow (a-b)^2(a+b) +(b-c)^2(b+c) +(c-a)^2(c+a) \geq 0 :\text{Đúng với mợi $a,b,c \geq 0$}$
Dấu $=$ khi $a=b=c$
Ta có (áp dụng nhận xét )
$\sqrt[3]{a^3+b^3} \geq \frac{a+b}{\sqrt[3]{4}}$
$ \sqrt[3]{c^3+8b^3} \geq \frac{c+2b}{\sqrt[3]{4}}$
$\sqrt[3]{8(a^3+b^3)+c^2} \geq \frac{2a+2b+c}{\sqrt[3]{9}} \geq \frac{6}{\sqrt[3]{9}}$
Vậy
$\sqrt[3]{a^3+b^3}+\sqrt[3]{c^3+8b^3}+\sqrt[3]{8(a^3+b^3)+c^3} \geq \frac{a+3b+c}{\sqrt[3]{4}} +\frac{6}{\sqrt[3]{9}}$
$\Rightarrow \sqrt[3]{a^3+b^3}+\sqrt[3]{c^3+8b^3}+\sqrt[3]{8(a^3+b^3)+c^3} \geq \frac{2a+2b+c}{\sqrt[3]{4}} +\frac{6}{\sqrt[3]{9}} $ :$\text{Do $b \geq a$}$
$\Rightarrow \sqrt[3]{a^3+b^3}+\sqrt[3]{c^3+8b^3}+\sqrt[3]{8(a^3+b^3)+c^3} \geq \frac{6}{\sqrt[3]{4}} +\frac{6}{\sqrt[3]{9}}$
Cộng Lại ta có :
$P \geq\frac{6}{\sqrt[3]{4}} +\frac{6}{\sqrt[3]{9}} +3$
Vậy $P_{min} = \frac{6}{\sqrt[3]{4}} +\frac{6}{\sqrt[3]{9}} +3$
Dấu $=$ xảy ra $\Leftrightarrow a=b=1 , c=2$

Điểm mở rộng 10

Bài viết đã được chỉnh sửa nội dung bởi E. Galois: 01-03-2013 - 17:43
Ghi điểm


#19
E. Galois

E. Galois

    Chú lùn thứ 8

  • Quản lý Toán Phổ thông
  • 3861 Bài viết
Trận đấu đã kết thúc. Mời các toán thủ nhận xét bài làm của nhau

1) Xem cách đăng bài tại đây
2) Học gõ công thức toán tại: http://diendantoanho...oạn-thảo-latex/
3) Xin đừng đặt tiêu đề gây nhiễu: "Một bài hay", "... đây", "giúp tớ với", "cần gấp", ...
4) Ghé thăm tôi tại 
http://Chúlùnthứ8.vn

5) Xin đừng hỏi bài hay nhờ tôi giải toán. Tôi cực gà.


#20
Nguyen Duc Thuan

Nguyen Duc Thuan

    Sĩ quan

  • Thành viên
  • 367 Bài viết
Bài của hoangtubatu995The gunners ghi nhầm GT là: $2a+2b+c=6$
Sau đây là đáp án chính thức:

Đề bài: Cho a,b,c>0 thoả mãn: $2a+2b+c\geq 6$ & $a\leq b$. Tìm giá trị nhỏ nhất của biểu thức:
$P=\sqrt{a^2+b^2}+\sqrt{c^2+4b^2}+\sqrt{4(a^2+b^2)+c^2}+a^4+b^3+c^2+b-2c$
Lời giải:
Ta có: $A^2+B^2\geq 2AB$
$\Rightarrow 2(A^2+B^2)\geq (A+B)^2$
Hay $A^2+B^2\geq \frac{(A+B)^2}{2}$ (1) (Dấu "=" xảy ra khi A=B)
Tương tự với 3 số ta cũng được: $A^2+B^2+C^2\geq \frac{(A+B+C)^2}{3}$ (2) (Dấu "=" xảy ra khi A=B=C)
Áp dụng (1) & (2) Và BĐT Cauchy vào P, ta có:
$P=\sqrt{a^2+b^2}+\sqrt{c^2+(2b)^2}+\sqrt{(2a)^2+(2b)^2+c^2}+(a^4+1+1+1)+(b^3+1+1)+(c^2+4)+b-2c-9$
$\geq \frac{a+b}{\sqrt{2}}+\frac{c+2b}{\sqrt{2}}+\frac{2a+2b+c}{\sqrt{3}}+4a+3b+4c+b-2c-9$
$= \frac{a+3b+c}{\sqrt{2}}+\frac{2a+2b+c}{\sqrt{3}}+2(2a+2b+c)-9$
$\geq 3\sqrt{2}+2\sqrt{3}+12-9=3+3\sqrt{2}+2\sqrt{3}$
(Chú ý rằng: $2a+2b+c\geq 6$ & $a\leq b$ thì suy ra $a+3b+c\geq 6$)
Vậy $Min(P)=3+3\sqrt{2}+2\sqrt{3}$ khi $a=b=1$ & $c=2$ $(\square)$






1 người đang xem chủ đề

0 thành viên, 1 khách, 0 thành viên ẩn danh